1answer.
Ask question
Login Signup
Ask question
All categories
  • English
  • Mathematics
  • Social Studies
  • Business
  • History
  • Health
  • Geography
  • Biology
  • Physics
  • Chemistry
  • Computers and Technology
  • Arts
  • World Languages
  • Spanish
  • French
  • German
  • Advanced Placement (AP)
  • SAT
  • Medicine
  • Law
  • Engineering
finlep [7]
3 years ago
15

somebody please help me on this it is for geometry please don’t waste my answers i’ll mark you the brainliest

Mathematics
1 answer:
vivado [14]3 years ago
6 0

Answer:

Smallest: XY

Second smallest: YZ

Biggest: XZ

Step-by-step explanation:

First, just observe the triangle,

see which one is the smallest,

if you just look, it would be XY

Next, the second smallest looks like YZ

Lastly, the last side is the largest if you just look at it

You might be interested in
Hello, Brainly community!
ioda

Answer:

(B)  \displaystyle \frac{W(3.1) - W(2.9)}{0.2}

General Formulas and Concepts:

<u>Calculus</u>

Limits

Derivatives

  • The definition of a derivative is the slope of the tangent line.

Derivative Notation

Instantaneous Rates

  • Tangent Line: \displaystyle f'(x) = \frac{f(b) - f(a)}{b - a}

Step-by-step explanation:

Since we are trying to find a <em>rate</em> at which W(t) changes, we must find the <em>derivative</em> at <em>t</em> = 3.

We are given 2 close answer choices that would have the same <em>numerical</em> answer but different <em>meanings</em>:

  1. (A)  \displaystyle  \lim_{t \to 3} W(t)
  2. (B)  \displaystyle \frac{W(3.1) - W(2.9)}{0.2}

If we look at answer choice (A), we see that our units would simply just be volume. It would not have the units of a rate of change. Yes, it may be the closest numerically correct answer, but it does not tell us the <em>rate</em> at which the volume would be changing and it is not a derivative.

If we look at answer choice (B), we see that our units would be cm³/s, and that is most certainly a rate of change. Answer choice (B) is also a <em>derivative</em> at <em>t</em> = 3, and a derivative tells us what <em>rate</em> something is changing.

∴ Answer choice (B) will give us the best estimate for the value of the instantaneous rate of change of W(t) when <em>t</em> = 3.

Topic: AP Calculus AB/BC (Calculus I/I + II)

Unit: Differentiation

Book: College Calculus 10e

8 0
3 years ago
Select the correct answer.<br> Find the value of g(7) for the function below.<br> g(x) = zur
ale4655 [162]

Answer:

B

Step-by-step explanation:

To evaluate g(7) substitute x = 7 into g(x) , that is

g(7) = \frac{7}{8} × 7 - \frac{1}{2}

      = \frac{49}{8} - \frac{4}{8}

      = \frac{45}{8} → B

5 0
3 years ago
A full moon occurs about every 30 days . If the last full moon occurred on a Friday, how many days will pass before a full moon
pentagon [3]
The answer is 30 days until it appears on a Friday again. In order to see the full moon again, we have to go through the entire cycle. So, it will be about 30 days before we would have seen another one. 
7 0
3 years ago
Evaluate the expression if x = - 8 , y = 7 , and z = - 11 . <br><br> - 11 - z =?
lyudmila [28]

Answer:

-11 - z

(-11) - (-11)

= 0

I hope I have helped.

5 0
2 years ago
Show That The Points(-1, -3), (-4, 7), (2,-13) Are Collinear!!!
GaryK [48]

Answer:

3 Quadrant

2 Quadrant

4 Quadrant

They are Collinear.

Please Mark As Brain list

6 0
2 years ago
Read 2 more answers
Other questions:
  • The _____ force always points opposite to the direction an object moves, or tries to move.
    6·2 answers
  • Jerry is trying to earn two hundred nine dollars for some new video Games. If he charges forty seven dollars in moves lawn,how m
    15·2 answers
  • Find the gcf of the monomial factor 15a-25b+20
    5·1 answer
  • NEED HELP ASAP!!
    10·1 answer
  • I need help with this simplify problem​
    13·1 answer
  • Determine the value of the expression: 5^4 × 5^8/5^6 Write your answer in exponential notation (in exponent form).
    5·1 answer
  • Does the point ( 1, 6 ) satisfy the equation <br><img src="https://tex.z-dn.net/?f=7x%20%5E%7B2%7D%20%20-%206x%20%2B%205" id="Te
    7·1 answer
  • Please help me- AND PLEASE EXPLAIN
    5·1 answer
  • Solve the question below
    7·1 answer
  • Find an equation in point-slope form for the line having the slope m= - 4 and containing the point (4,5).
    11·1 answer
Add answer
Login
Not registered? Fast signup
Signup
Login Signup
Ask question!